PA Easy- Endocrine

अब Quizwiz के साथ अपने होमवर्क और परीक्षाओं को एस करें!

Patients diagnosed with type 2 diabetes are encouraged to maintain their plasma glucose, to prevent or slow the initiation of chronic complications. What is the maximum level that is within ADA guidelines for a one to two hour postprandial plasma glucose in these patients? A 70 mg/dL B 100 mg/dL C 120 mg/dL D 180 mg/dL E 220 mg/dL

D 180 mg/dL The correct choice is D, 180 mg/dL. Patients are encouraged to adhere to lifestyle and medication treatment plans in order to help achieve this goal. Other ADA guidelines include maintaining the hemoglobin A1c < 7% or as close to normal (<6%) as possible, blood pressure < 130/80, and preprandial glucose between 90 and 130 mg/dL.

An adult patient was recently diagnosed with type 2 diabetes mellitus. She met with her diabetic educator to discuss suggested changes to her diet and exercise regimens. What percent of her total daily calories should be in the form of carbohydrates? A 5% to 15% B 20% to 25% C 35% to 40% D 45% to 65% E 70% to 80%

D 45% to 65% The correct choice is D, 45% to 65%. The American Diabetes Association recommends a diet with 45% to 65% carbohydrates, 25% to 35% fats, and 10% to 35% protein.

A 30-year-old female presents to your office for a routine physical exam. She has not seen a health care provider in many years. Upon talking with the patient, you find out that she had been diagnosed with hypertension several years ago, but was unable to afford the antihypertensive medications that were prescribed to her. She has no complaints at this time. Upon exam of the head and neck, you note widened spaces between her lower incisor teeth and a large, fleshy nose. Her skin is oily and she demonstrates mild proximal muscle weakness. Her EKG reveals a left axis deviation and widened QRS. What is the most likely rationale for her clinical presentation? A Diabetes mellitus B Cushing's syndrome C Hypothyroidism D Acromegaly E Clinical depression

D Acromegaly The correct choice is D, acromegaly. Patients with acromegaly have an abundance of growth hormone secretion. This leads to excessive growth of many areas of the body including soft tissue. Patients with acromegaly also have an increased incidence of hypertension and left ventricular hypertrophy. None of the other choices will cause this patient's constellation of symptoms. Patients with many endocrine disorders may develop weaknesses as seen in this patient, but the large nose and widely spaced teeth are characteristic of acromegaly.

To which of the following areas does follicular thyroid cancer most commonly first spread? A Intraglandular metastasis B Local spread into regional vocal cords C Distant lymph nodes D Bone and lung via bloodstream E Local extension into the muscle and trachea

D Bone and lung via bloodstream The correct choice is D, bone and lung via bloodstream. Follicular thyroid cancer can spread to regional lymph nodes and distant sites via the blood stream. Choice A, intraglandular metastasis, is seen more commonly in papillary thyroid carcinoma, and choice B, local spread into regional vocal cords, occurs in anaplastic thyroid carcinoma. Choice C, distant lymph nodes, and choice E, local extension into the muscle and trachea, are more commonly seen in patients with medullary thyroid cancer.

A 12-month-old infant is being worked up for congenital growth hormone deficiency. Her length at birth was short and she has been consistently less than the third percentile on her height chart. Her past medical history is only significant for three seizures since birth. Upon exam you note that she has a full face and her body proportions are normal. The remainder of the exam is normal. What is the most likely cause of her seizures? A Abusive head trauma B Brain tumor C Arteriovenous malformation D Hypoglycemia E Cerebrovascular accident

D Hypoglycemia The correct choice is D, hypoglycemia. Patients with growth hormone deficiency will not have the counter regulatory action of growth hormone against insulin. This will allow unregulated insulin action causing hypoglycemia and possible associated seizures. Choices A, abusive head trauma, B, brain tumor, C, arteriovenous malformation, and E, cerebrovascular accident, can all cause seizures. This patient does not show any signs or symptoms of any of these disorders.

A patient was recently diagnosed with type 1 diabetes mellitus. A treatment plan was initiated, with a combination regimen of insulin. Which of the following types of insulin works well with a rapidly acting insulin, such as insulin lispro, to provide 24-hour coverage for the patient? A NPH insulin B Regular insulin C Insulin aspart D Insulin glargine E Humalog 75/25

D Insulin glargine The correct choice is D, insulin glargine. This is the only long acting insulin listed. The combination of a long acting insulin with a rapidly acting insulin provides physiologic insulin replacement to the patient. This regimen provides postprandial control after meals and basal coverage throughout the day and night. Choice A NPH insulin, can be used by itself in two or more injections throughout the day. Choice B, regular insulin, can be used instead of rapid acting insulin, and not in combination with it. Choice C, insulin aspart, is a type of rapidly acting insulin and would not be used in combination with another rapidly acting insulin. Choice E, Humalog 75/25, is a combination insulin preparation with 75% intermediate acting insulin and 25% insulin lispro.

A woman presents for a routine post-partum checkup four weeks after delivery of her child. She is currently breast feeding without difficulty. She tells you that she has been feeling anxious and very warm, despite the change of season into winter. She is happy as a mother and has not had problems caring for her baby. What is the next best step in the investigation of her symptoms? A Radioactive iodine uptake test B Serum total T4 level C Thyroid fine needle aspiration D Serum TSH level E MRI of the anterior pituitary

D Serum TSH level

A mother expresses concern for her teen son after feeling a lump in his neck. He has no history of trauma to his neck. Surgical history is negative, and the patient does not take any medications. The mother tells you that thyroid problems run in the family. The patient has not been ill recently. Upon exam you feel a nontender, firm nodule on the right side of his thyroid with associated cervical lymphadenopathy. His serum TSH level is within the reference range. Radionuclide thyroid scanning demonstrates a "cold" nodule in the right side of the thyroid. What is the most appropriate next step in the work up of this patient? A MRI of the anterior pituitary B CT of the thyroid C MRI of the thyroid D Thyroid nodule fine needle aspiration E Emergent thyroidectomy

D Thyroid nodule fine needle aspiration The correct choice is D, thyroid nodule fine needle aspiration. With the advent of fine needle aspiration, it has become much easier, safer, and more reliable to obtain a specimen for biopsy. This patient has several characteristics that increase his risk of malignancy including his gender, young age, firmness of the nodule, and related lymphadenopathy. These, along with the ease of biopsy, suggest this path for diagnostic work up. Not enough information is known to warrant an emergent thyroidectomy, choice E. Choice A, MRI of the anterior pituitary, would be warranted if there was a suspicion of a pituitary cause of the thyroid nodules. Since the TSH is normal and the patient is not presenting with headaches or other pituitary related symptoms, this is not suggested. Choice B, CT of the thyroid, and choice C, MRI of the thyroid, would not provide any additional information after the thyroid scan. They may be helpful prior to any surgery if needed.

A 60-year-old man presented with a mass in the left lobe of the thyroid. Fine needle aspiration was consistent with papillary carcinoma. There was no evidence of locally invasive or metastatic disease. Which of the following treatments is recommended for this patient? A Chemotherapy B External beam radiation C Preoperative radioiodine ablation D Total thyroidectomy

D Total thyroidectomy Papillary carcinoma is the most common type of thyroid malignancy. Treatment includes a thyroid lobectomy and isthmusectomy or total thyroidectomy. The decision regarding the extent of the surgery is based on the extent of the disease, the tumor size, and histiologic grade. A poor prognosis is seen in males, patients older than 50 years of age, primary tumors greater than 4 cm in size, tumors that are less well differentiated, or evidence of locally invasive or metastatic disease. Accordingly, the recommended treatment for this patient is a total thyroidectomy. Radioiodine ablation is recommended postoperatively.

Your patient with type 2 diabetes mellitus (T2DM) who is already on metformin and pioglitazone but continues to have elevated fasting and postprandial blood sugars. Together, you elect to begin insulin glargine at 10 U QHS initially and have the patient self manage their upwards titration adding 1 IU SQ in the evening every day that their morning fasting glucose is above 100 mg/dL. You caution the patient that she should expect to take approximately how long to achieve well-controlled sugars? A immediately B about one week C about two weeks D about one month E about three months

D about one month There are many regimens to begin and titrate basal insulins. Regardless, most patients require approximately 45 units of glargine to reach therapeutic goals. If they began glargine at 10 IU and add 1 IU daily until they reach 45 IU, this should take approximately 35 more days, or approximately one month.

A 38-year-old male complains of increasing fatigue, weakness, weight loss, and intermittent nausea, vomiting, and diarrhea over the past few months. He has noted some agitation at times. When this first started he thought that this was related to a virus but the GI symptoms have reoccurred on multiple occasions. Labs show a complete blood count (CBC) within the normal reference range. He is noted to have hyponatremia. On examination you note the multiple areas of hyperpigmentation as seen below. His blood pressure in the office is 100/50, P = 66, T = 97.1˚F. What test would you order to confirm your suspected diagnosis? A dexamethasone suppression test B vasopressin challenge test C radioactive iodine uptake scan D cosyntropin stimulation test E follicular stimulation test

D cosyntropin stimulation test The correct answer is (D). The patient's symptoms and examination findings are consistent with a diagnosis of Addison's disease, which is most likely due to an autoimmune process that destroys the adrenal glands resulting in a chronic adrenal insufficiency. The cosyntropin (ACTH) stimulation test should reveal a low am cortisol level and an elevated ACTH level if he has Addison's disease. The dexamethasone suppression test, choice (A), is a laboratory test for Cushing's syndrome. The vasopressin challenge test, choice (B), is a laboratory test for diagnosis of diabetes insipidus. A radioactive iodine uptake scan, choice (C), is used in the diagnosis of thyroid disease (hyperthyroidism and thyroid nodules). A follicular stimulation test, choice (E), is a factitious test.

When used for advanced carcinoma of the prostate, chronic administration of leuprolide inhibits the synthesis of androgens by _____________________ A blocking gonadotropin-releasing hormone (GnRH) receptors at the anterior pituitary B blocking luteinizing hormone (LH) receptors on interstitial (Leydig) cells of the testes C increasing the secretion of GnRH from the hypothalamus D inhibiting pulsatile secretion of gonadotropins from the anterior pituitary E upregulation of the number of GnRH receptors at the anterior pituitary

D inhibiting pulsatile secretion of gonadotropins from the anterior pituitary Leuprolide is a GnRH (LHRH) agonist that suppresses the pulsatile secretion of follicle stimulating hormone (FSH) and LH (gonadotropins) from the anterior pituitary when given chronically. Continuous administration of a GnRH agonist causes down-regulation of GnRH receptors on gonadotropes, which, in turn suppresses gonadotropin release and gonadal function. Decreased amounts of LH, in particular, lead to diminished production of androgens by the testes (especially DHT), which support prostate growth. It is believed that by interrupting the hormonal pathways that modulate prostatic growth, tumor development and metastasis is slowed.

A 56-year-old woman is being seen for regular assessment and monitoring of her type 2 diabetes mellitus. She has been following a strict diet and exercise plan for 2 years with the addition of metformin 6 months ago for an increased HgA 1c level. Her HgA 1c at today's visit is 7.1. What is the appropriate management for this patient? A add exenatide to her current therapy B change her oral therapy to rosiglitazone C add insulin to her current therapy D maintain her current therapy and recheck in 6 months

D maintain her current therapy and recheck in 6 months The HgA 1c goal for this patient is less than 6.5, with action at a level of greater than 8.0. The appropriate action at this time is to continue her current therapy and reassess in 6 months.

A 28-year-old woman who was born and brought up just outside of Washington, DC, comes in for evaluation of vague "problems with swallowing." She has no other symptoms except "my neck is bigger than it used to be." Examination reveals only a diffuse, somewhat irregular, nontender enlargement of the thyroid gland with distinct masses palpable within it. What is the most likely diagnosis? A endemic goiter B Graves disease C Hashimoto thyroiditis D multinodular goiter E thyroid carcinoma

D multinodular goiter Multinodular goiter is the most likely in a woman with these findings in the United States. It may be nontoxic as in this case or toxic, i.e., producing excessive thyroid hormones which cause symptoms of hyperthyroidism. Endemic goiter (A), which may present as a simple enlargement of the thyroid or as a multinodular one, is found almost entirely in iodine-deficient areas of the world and is extremely rare in the U.S. In Graves disease (B), the thyroid is enlarged and may exhibit a thrill and a bruit. In addition, the patient would have other signs of hyperthyroidism. The thyroid in Hashimoto thyroiditis (C) is diffusely enlarged and firm with fine nodules. A thyroid carcinoma (E) usually presents as a firm, nontender nodule in the gland.

A 42-year-old male comes to the office to discuss his total cholesterol of 215 mg/dl, which was obtained last week at a health fair. He is generally healthy, but smokes one pack of cigarettes per day for the past 10 years. He is unsure of his family history. What is your next step? A prescribe simvastatin B advise dietary changes C repeat the total cholesterol today, fasting D obtain a fasting lipid profile E prescribe gemfibrozil

D obtain a fasting lipid profile The correct answer is (D). The patient comes in only with total cholesterol. With this information a fasting lipid profile is indicated to evaluate the patient's cardiovascular risk and should be the next step. Repeating the total cholesterol is not very helpful. It is premature to recommend a specific treatment at this point until you fully assess the patient's risk.

A 45-year-old man with a history of neck irradiation for Hodgkin lymphoma at the age of 15 is found to have a 1.5-cm, nontender, firm thyroid nodule. Upon laboratory evaluation, the patient is found to be euthyroid, and fine needle biopsy reveals malignancy. What histologic type is most likely? A anaplastic B follicular C medullary D papillary

D papillary Thyroid carcinoma often presents as an asymptomatic thyroid nodule. The most common histologic form is papillary carcinoma, representing more than 80% of cases.

A 29-year-old woman comes to the office because she "just keeps gaining weight and can't stop." Since she was last seen in the office at age 24, she has gained nearly 100#. Physical examination is remarkable for blood pressure of 140/92. She appears depressed. Her trunk and abdomen are heavy with normal-sized extremities. Her facial hair is dark and in a "male" distribution. Large dark violaceous striae are present on her abdomen and proximal extremities. Which of the following additional findings is most consistent with this presentation? A Buffalo hump B Doughy, thickened skin C Exophthalmos D Lid lag E Thickened tongue

A Buffalo hump This woman appears to have Cushing Syndrome (hypercortisolism) which is characterized, in addition to the signs listed above, by plethoric facies, supraclavicular fatpads, and the so-called "buffalo hump." Doughy, thickened skin (B) and thickened tongue (E) may be found in hypothyroidism, exophthalmos (C) in Graves disease, and lid lag (D) in hyperthyroidism from any cause.

A 55-year-old patient presents with symptoms of polyuria and is worried that he may have diabetes mellitus. His past medical history includes hypertension and lung cancer. His family history is negative for diabetes mellitus. He works as an accountant and has no history of head trauma. He has an average diet and fluid intake. His fasting plasma glucose is 110 mg/dL and his hemoglobin A1c is 5%. What is the most likely cause of his polyuria? A Cancer related diabetes insipidus B Type 2 diabetes mellitus C Nephrogenic diabetes insipidus D Familial hypothalamic diabetes insipidus E Primary polydipsia

A Cancer related diabetes insipidus The correct choice is A, cancer related diabetes insipidus. Solid tumors, such as cranipharyngiomas and metastatic lung and breast cancer, are known to cause diabetes insipidus. This patient has a history of lung cancer. Choice B, diabetes mellitus, is not present in this patient. His pasting glucose and hemoglobin A1c are within the reference range. Choices C, nephrogenic diabetes insipidus, and choice D, familial hypothalamic diabetes indipidus, are causes of polyuria. Both are noted in young patients. Choice E, primary polydipsia, is associated with excessive drinking of liquids, which leads to excessive urination. There is no history of this occurring in this patient.

A 9-year-old girl is brought to the office by her mother who has noticed that the child's neck looks "odd." The girl states that it feels "like something is there," indicating the base of the neck, but that she has no difficulty swallowing. A thorough examination reveals only a firm, nontender, and diffusely enlarged thyroid with no thrills or bruits. All parameters on a thyroid function panel are within normal limits. Low levels of thyroid antibodies are present. What is this child's most likely diagnosis? A Chronic lymphocytic thyroiditis (Hashimoto thyroiditis) B Graves disease C Papillary carcinoma of the thyroid D Pituitary tumor E Subacute thyroiditis (deQuervain thyroiditis)

A Chronic lymphocytic thyroiditis (Hashimoto thyroiditis) Hashimoto thyroiditis, the most common thyroid disorder in the United States, is an autoimmune disease. In children, girls 8-15 years of age are most commonly affected. Graves disease is most common in women ages 20-40. The thyroid enlargement is diffusely enlarged but may be asymmetric and may be accompanied by a bruit. In addition, the patient will have signs and symptoms of hyperthyroidism. Thyroid antibodies and hormones are elevated and TSH suppressed. Thyroid carcinoma is also more likely in women and papillary carcinomas are the post common. They present as a firm nodule in the gland, most often between 10 and 40 years after exposure to radiation. Thyroid-hormone-secreting pituitary tumors (rare) usually present with signs and symptoms of hyperthyroidism without enlargement of the gland and normal or high serum TSH levels. DeQuervain thyroiditis presents with signs and symptoms of hyperthyroidism and an enlarged tender gland.

A 1-year-old adopted Chinese infant is brought into the pediatrician's office for a routine visit. The parents are concerned because they think the child is short for her age. They do not have very much information relating to the birth of the child, other than that she was considered slightly short at birth and she had a seizure at one month of age. The foster parents of the child were not concerned, since the infant was somewhat chubby and ate well. She has no history of hypotonia. Upon exam you note normal shaped eyes with mild nystagmus. The infant is less than the third percentile for weight and her limbs are in normal proportion to her height. Her lung and heart exam are normal. CBC is normal. What is the most likely cause of her short stature? A Congenital growth hormone deficiency B Sickle cell anemia C Prader-Willi syndrome D Osteochondrodysplasia E Cystic fibrosis

A Congenital growth hormone deficiency The correct choice is A, congenital growth hormone deficiency. Parents of children with this disorder typically become concerned when the child is between one and two years of age. This patient fits the characteristic picture of this disorder, with short stature, increased fat mass, and hypoglycemia due to relatively unopposed insulin action. This may have been the cause of her seizure. She may also have optic hypodysplasia as a cause of her nystagmus. Choice B, sickle cell anemia, can be associated with short stature. However, the CBC from this patient was normal. Choice C, Prader-Willi syndrome, is also associated with short stature. Children with this disorder commonly have almond shaped eyes and hypotonia at birth. Choice D, ostrochondrodysplasia, is a type of inherited skeletal abnormality that presents with short stature and abnormal body proportions. This patient doesn't have abnormal body proportions. Choice E, cystic fibrosis, can be associated with growth abnormalities, but this patient has no evidence of lung disease.

Which of the following types of thyroid cancer involve the parafollicular cells (or C cells) of the thyroid? A Medullary B Papillary C Follicular D Lymphoma E Anaplastic

A Medullary The correct choice is A, medullary thyroid carcinoma. Unlike papillary (choice B) and follicular (choice C) thyroid cancers, which arise from thyroid epithelial cells, medullary thyroid cancer arises from the parafollicular cells of the thyroid. Anaplastic thyroid cancer, choice E, arises from undifferentiated cells. Thyroid lymphoma, choice D, is most commonly of B cell follicle center cell origin.

A patient was prompted to visit his health care provider after his wife started to notice that he was not interested in eating, has lost weight, and has been suffering from nausea for the last few weeks. The practitioner notes hyperpigmentation of the patient's skin, although the patient denies any recent sun or tanning salon exposure. Routine non-fasting blood work reveals the following: Sodium = 130 meq/L Potassium = 5.2 meq/L Chloride = 105 meq/L Glucose = 135 mg/dL Hemoglobin = 13.0 g/dL Hematocrit = 39.0 WBC count = 8,000/mm 3 Which of the following physical exam findings would you expect to see in this patient? A Orthostatic hypotension B Wide, purple striae C Central obesity D Full facial features E Exophthalmous

A Orthostatic hypotension The correct choice is A, orthostatic hypotension. The first step in the discussion of this patient is the suspected diagnosis of adrenal insufficiency. Patients with this disorder will have an excess of ACTH, which will act like melanocyte stimulating factor on the skin and cause hyperpigmentation. In adrenal insufficiency, aldosterone is deficient, thereby causing a decrease in sodium retention and potassium excretion. Hypotension is found in approximately 90% of these patients, sometimes associated with syncope as well. Choices B, C, and D are found in patients with cortisol excess. Choice E can be found in patients with Graves' disease.

A patient seen at the prenatal clinic develops Graves disease at 25 weeks' gestation. Which of the following is the most appropriate treatment? A PTU 100 mg po tid B methimazole 10 to 30 mg po qd C propranolol 80 mg po qid D radioactive iodine therapy (RAI, 131I) E levothyroxine 0.1 mg po qd

A PTU 100 mg po tid In nonpregnant patients, PTU and methimazole are the drugs of choice for the management of Graves disease. During pregnancy, PTU has a lower incidence of crossing the placental barrier than does methimazole. It also is excreted into breast milk to a lesser degree than is methimazole. Propranolol will help with the symptoms of Graves but not treat it. It can also cause low birth weight in the infant. RAI is contraindicated in pregnancy. Levothyroxine will worsen a Graves patient's hyperthyroidism.

A patient presents with a several year history of developing coarse facial features, prognathism, widely spaced teeth, and sleep apnea. Upon exam, you also note oily skin and high blood pressure. What is the most likely cause of this patient's symptoms? A Pituitary adenoma B Pheochromocytoma C GHRH secreting hypothalamic tumor D Adrenal carcinoma E Ectopic ACTH secretion

A Pituitary adenoma The correct choice is A, pituitary adenoma. The patient is presenting with classic signs and symptoms of acromegaly. This disorder is caused most commonly by a growth hormone secreting pituitary adenoma. On rare occasions, it has been caused by choice C, a GHRH secreting hypothalamic tumor. Choice B, pheochromocytoma, is a tumor of the adrenal glands that causes high blood pressure, but does not cause the classic body features of acromegaly. Choice D, adrenal carcinoma, and choice E, ectopic ACTH secretion, may cause Cushing's syndrome, but not acromegaly.

A 60-year-old female presents to her primary care practitioner for her yearly check up. Her past medical history is significant only for a history of repeat kidney stones. She enjoys gardening outside, has no significant family history, and is not taking any medications. Routine mammograms have been normal. During the review of her systems, the patient describes feeling tired lately and has noticed muscle aches over the last few weeks. Initial lab values are shown below: Hemoglobin= 12.0 gm/dL Hematocrit = 36% BUN= 18 mg/dL Creatinine= 1.0 mg/dL Calcium= 12 mg/dL Phosphate= 2.0 mg/dL Intact PTH= 80 pg/mL TSH= 3.0 uU/L What is the most likely pathology associated with the findings in this patient? A Solitary parathyroid adenoma B Excessive Ingestion of calcium C Parathyroid carcinoma D Medullary thyroid cancer E Chronic renal disease

A Solitary parathyroid adenoma The correct choice is A, solitary parathyroid adenoma. Primary hyperparathyroidism is caused by a solitary adenoma in 80% of patient cases. Choices B, D, and E do not cause primary hyperparathyroidism. Choice C can cause primary hyperparathyroidism, but with an incidence of less than 1%.

A 40-year-old female presents to the clinic with symptoms of a 10 pound unintentional weight loss, diarrhea, and palpitations. She has a positive family history for thyroid disease and diabetes mellitus. Her screening TSH level is 0.15mIU/L, and her fasting plasma glucose is 105mg/dL. Which physical exam finding would you expect to find in this patient? A Tremor B Thin hair C Puffy face D Delayed reflexes E Bradycardia

A Tremor The correct choice is A, tremor. This question first requires the identification of hyperthyroidism in the patient. The symptoms of weight loss, diarrhea, and palpitations, with a low TSH level, are characteristic of the disorder. Hyperthyroidism also presents most commonly in women in the third and fourth decade, as well as in the elderly. Common physical exam findings of hyperthyroidism include tachycardia, warm moist skin, tremor, goiter, muscle weakness, lid retraction or lag, and gynecomastia. Choices B, thin hair, C, puffy face, D, delayed reflexes and E, bradycardia, are all signs of hypothyroidism.

A 10-year-old girl was recently diagnosed with type 1 diabetes. She appears to be adjusting well to her regimen of insulin and glucose monitoring, and follows a careful meal plan that she and her mother work on together. At a follow-up visit, she says that she will "die" if she can't participate on her summer swim team, but her mother is clearly concerned about her becoming hypoglycemic if she exercises that vigorously. In addition to reminding them that regular aerobic exercise is an important part of her overall well-being, which of the following is the most appropriate advice for them? A add an extra snack before exercising to the diet plan B ask the coach for a less-rigorous practice regimen C avoid dextrose-containing beverages during exercise D increase the amount of insulin on practice and competition days E try a less energy-intensive sport than swimming

A add an extra snack before exercising to the diet plan Appropriate advice for children with type 1 diabetes is to have extra snacks before exercise. In addition, they should drink dextrose-containing fluids (C) during exercise and decrease insulin (D), while monitoring glucose before, during, and after exercise. Exercising less rigorously (B) and avoiding any particular sport (E) is usually not necessary with careful management of the diabetes.

A 45-year-old woman presents with weight gain, fatigue, dry skin, constipation, and oligomenorrhea. On physical exam, bradycardia and slow deep tendon reflexes are noted. Her free T 4 is low and TSH is elevated. Which of the following medications may be responsible for her condition? A amiodarone B beta-blockers C levadopa D hydrocholorthiazide

A amiodarone Hypothyroidism is reported in up to 10% of patients taking amiodarone, an antiarrhythmic medication. With the high iodine content of the medication and the structural similarities to thyroxine, thyroid abnormalities occur. Common side effects of amiodarone include bradycardia and constipation, so laboratory evaluation for thyroid dysfunction must be used.

A 26-year-old woman has decreased appetite, weight gain, cold intolerance, hoarse voice, constipation, and arthralgias. What is the most likely etiology of her condition? A autoimmune thyroiditis B congenital hypothyroidism C dietary iodine deficiency D surgical resection of the thyroid gland

A autoimmune thyroiditis Auto immune thyroiditis is the most common cause of hypothyroidism in the United States. Dietary iodine deficiency is the most common cause in underdeveloped regions of the world.

A 62-year-old obese woman presents with progressive numbness and tingling in her feet for the past 3 months. On physical examination, the patient is found to have decreased sensation to pinprick and vibration, absence of ankle reflexes, and difficulty with tandem walking. Which is the most common etiology of her symptoms? A diabetes mellitus B alcoholism C vitamin B12 deficiency D spinal cord tumor E rheumatoid arthritis

A diabetes mellitus Peripheral neuropathy is a syndrome that is manifested by muscle weakness, paresthesias, decreased deep tendon reflexes, and autonomic disturbances most commonly in the hands and feet, such as coldness and sweating. There are many causes of peripheral neuropathy ranging from metabolic conditions to malignant neoplasm, rheumatoid arthritis, and drug and alcohol use. The increase in non-insulin-dependent diabetes mellitus due to obesity in the American population has increased the incidence of associated disease states.

A 76-year-old man, is brought to the emergency department by his niece after she found him wandering around his yard in the cold wearing only a tee shirt and jeans. When she set up his pill container about 36 hours earlier, he seemed his usual self but, in retrospect, possibly a little more confused than usual. The niece says that he has "high blood," treated with a "white fluid pill," "sugar diabetes," treated with an oral medication, and early "old timer's" dementia treated with "a memory pill." Vital signs include an oral temperature of 100.8F, pulse 100 beats per minute, respirations 24 and somewhat shallow, and blood pressure of 88/52. Initial examination reveals a slightly dehydrated, stuporous man appearing older than his stated age, who smells strongly of urine. He has no lateralizing signs. What is the most likely cause of the mental status changes? A hyperglycemic hyperosmolar state B lactic acidosis C stroke D urinary tract infection E worsening dementia

A hyperglycemic hyperosmolar state The combination of confusion and dehydration in a patient with diabetes type 2 who is taking a diuretic strongly suggest hyperosmolar state. Patients with lactic acidosis (B) have marked hyperventilation and, usually, signs and symptoms of a serious illness. The lack of lateralizing signs makes a stroke (C) less likely. Urinary tract infection (D) could certainly cause confusion and incontinence in an elderly man and should be investigated. Alzheimer dementia (E) progresses slowly; sudden decompensation is usually due to delirium.

A 5-year-old girl is seen in your office with a several week history of increased thirst, weight loss, and blurred vision. She has a positive family history for diabetes mellitus, hypertension, and stroke. Her urine dipstick chemical testing reveals positive glucose and negative ketones, protein, blood, and nitrites. Which of the following laboratory test results would support a diagnosis of diabetes mellitus in this patient? A random plasma glucose > 200 mg/dL B random urine glucose dipstick > 1+ C plasma hemoglobin A1c < 7% D fasting plasma glucose > 110 mg/dL E 2-hour postprandial plasma glucose > 135 mg/dL

A random plasma glucose > 200 mg/dL The correct choice is A, random plasma glucose > 200 mg/dL. The most recent recommendations from the international committee of diabetes experts list the following as diagnostic criteria, which must be confirmed before use: 1) Symptoms of diabetes and a random plasma glucose > 200 mg/dL 2) Fasting plasma glucose > 126 mg/dL 3) Two-hour plasma glucose > 200 mg/dL during a standard 75 gram oral glucose tolerance test More recently, the American Diabetes Association has also included a hemoglobin A1c level equal to or greater than 6.5% as a diagnostic criteria as well. None of the other choices fit into this list of diagnostic criteria.

A patient was recently diagnosed with acromegaly. He was reading on the Internet that acromegaly is associated with diabetes mellitus. He is especially concerned, since his father was recently diagnosed with diabetes as well. How would you reply to the patient's following question: "What percentage of people with acromegaly develop diabetes mellitus?" A 10% B 25% C 50% D 65% E 80%

B 25% The correct choice is B, 25%. Growth hormone is a counter-regulatory hormone of insulin, and therefore acts against insulin. This can lead to hyperglycemia, glucose intolerance, and diabetes mellitus in 25% of patients with acromegaly.

A teenage girl presents to the emergency department with her parents. She has had symptoms of a urinary tract infection for the last two days, but did not tell her parents until today. She is not sexually active. Today, the girl also has diffuse abdominal pain with vomiting, general malaise, and difficulty breathing. She has no significant past medical history. Her physical exam reveals sinus tachycardia and deep fast respirations with no localization of abdominal pain or rebound tenderness. Initial lab test results reveal a plasma glucose = 378 mg/dL and serum bicarbonate = 14 mEq/L. What is the most likely diagnosis? A Hyperosmolar nonketotic hyperglycemia B Diabetic ketoacidosis C Chronic corticosteroid use D Gestational diabetes E Schmidt syndrome

B Diabetic ketoacidosis The correct choice is B, diabetic ketoacidosis (DKA). Classic signs and symptoms of this disorder include polyuria, polydipsia, marked fatigue, nausea, vomiting, signs of dehydration, fruity breath odor, postural hypotension, Kussmaul respirations, and possibly mental stupor or coma. Patients with type 1 diabetes mellitus may present for the first time in DKA. DKA is commonly precipitated by a recent infection. Choice A, hyperosmolar nonketotic hyperglycemia, is seen in patients with type 2 diabetes and presents with extremely high plasma glucose levels without acidosis and ketosis. Choice C, chronic corticosteroid use, can cause hyperglycemia and possible glucose intolerance or diabetes mellitus. This patient has no history of oral corticosteroid use. Choice D, gestational diabetes, occurs in women who are pregnant. Choice E, Schmidt syndrome, is an autoimmune polyglandular syndrome which includes diabetes mellitus in greater than 50% of patients.

A 50-year-old woman presents with a history of polyuria, polydipsia, muscle aches, bone pain, nausea, and constipation for the past few months. Her past medical history is significant for a pituitary adenoma, peptic ulcer disease, and kidney stones. You are considering a diagnosis of primary hyperparathyroidism. Which of the following lab values would you expect in this patient? A Low serum calcium and elevated serum PTH B Elevated serum calcium and PTH levels C Elevated serum calcium and low serum PTH levels D Low serum phosphate and PTH levels E Elevated serum phosphate and calcium levels

B Elevated serum calcium and PTH levels The correct choice is B, elevated serum calcium and PTH levels. In primary hyperparathyroidism, excess PTH is secreted and stimulates a rise in serum calcium by increasing calcium release from bone, thereby reducing renal clearance of calcium and increasing calcium absorption through the intestine. In choice A, low serum calcium is not seen in hyperparathyroidism. In choice C and D, low serum PTH levels aren't seen in hyperparathyroidism. In choice E, elevated serum phosphate levels aren't seen in hyperparathyroidism. It typically presents with low serum phosphate levels.

A 35-year-old woman comes to see you with symptoms of anxiety, sweating, and tremors. She has no history of diabetes, liver or kidney failure, hormone deficiencies, or past surgeries. She states that she is not currently prescribed any medications and does not drink alcohol. She is concerned that she will not be able to continue to care for her husband, who has a long history of diabetes mellitus. She is asking you to give her some medication to stop her symptoms. Initial lab results are as follows: Plasma glucose = 54 mg/dL (70-110 mg/dL) TSH = 2.0 mIU/L (0.34-4.25 uIU/mL) Insulin = 35 uU/mL (2.0-20 uU/mL) C-peptide= 0.4 ng/mL (0.5-2.0 ng/mL) Her symptoms are relieved with the drinking of orange juice. What is the most likely cause of her hypoglycemia? A Alimentary hypoglycemia B Factitious hypoglycemia C Beta cell insulinoma D Congenital hyperinsulinism E Reactive hypoglycemia

B Factitious hypoglycemia The correct choice is B, factitious hypoglycemia. This occurs when patients accidentally or on purpose self-administer insulin or an insulin secretagogue. This occurs most commonly among health care personnel, patients with diabetes or family members of those with diabetes, and people with a history of other factitious diseases. It can also happen secondary to a pharmacy error. Patients with this disorder will have increased measured insulin without the physiologic corresponding increase in C-peptide. Choice A, alimentary hypoglycemia, is a cause of hypoglycemia in patients with a history of gastrectomy. Choice C, beta cell insulinoma would present with elevated levels of both insulin and C-peptide. Choice D, congenital hyperinsulinism would have presented itself earlier than in a patient who is 35 years old. Choice E, reactive hypoglycemia, occurs after eating a meal and must be documented in this fashion.

Patients prescribed which of the following medications may present with a dorsocervical fat pad, thin extremities, and central obesity, which may mimic Cushing's syndrome? A ACE inhibitors B HIV antiretrovial medications C Loop diuretics D Opioid analgesics E Aminoglycosides

B HIV antiretrovial medications The correct choice is B, HIV antiretroviral medications. Patients using highly active antiretroviral therapy (HAART) can develop partial lipodystrophy and changes in body fat distribution. This complication occurs more commonly with long-term use, and affects 1/3 to 2/3 of patients on therapy for longer than one year. The other medication classes noted are not associated with lipodystrophy. Opioids, choice D, are associated with a decline of cortisol secretion.

A 60-year-old female patient living in New York City presents for a routine office visit prior to travelling to Europe with her husband for a second honeymoon. Her medical history includes pernicious anemia, for which she is being treated with vitamin B12 supplementation. She uses no other medications. Past surgical history includes an appendectomy at age five. Upon reviewing her symptoms, you find that she has been trying to self treat for chronic constipation, without positive results. She also notes that she is easily tired and has gained 10 pounds in the last few months. On exam you note bradycardia and cool, dry skin. What is the most likely cause of her recent symptoms? A Major depression B Hashimoto's thyroiditis C Hypothalamic destruction D Toxic thyroid nodule E Iodine deficiency

B Hashimoto's thyroiditis The correct choice is B, Hashimoto's thyroiditis. This autoimmune disorder is by far the most common cause of hypothyroidism in adults. This patient is presenting with classic signs and symptoms of hypothyroidism including bradycardia, cool, dry skin, and modest weight gain. She also has a history of another autoimmune disorder, pernicious anemia, which is commonly associated with hypothyroidism. Choice A, major depression, seems unlikely since the patient is planning a second honeymoon. Although many of her symptoms can be attributed to depression, her cool, dry skin is not characteristic of the disorder. Choice C, hypothalamic destruction, can cause hypothyroidism, but it is rare. Patients with choice D, toxic thyroid nodule, would typically present with symptoms of hyperthyroidism. Choice E, iodine deficiency, is a cause of hypothyroidism, but it is unlikely in patients living in developed countries with access to a variety of food sources.

A 32-year-old female with history of non-insulin-dependent diabetes mellitus (NIDDM) returns to the clinic for a routine visit. She reports that she her blood sugars have been controlled. However, her vitals today show a blood pressure (BP) of 136/78 P = 72. You note the past three office visits that her blood pressures were in the mid 130s to 140 systolic. She is not currently taking anything for her blood pressure but admits that she has not been exercising as much as usual. What do you advise the patient? A Advise lifestyle modification and continue to monitor BP next visit. B Initiate antihypertensive therapy to target BP < 130/80. C Initiate DASH diet and recheck blood pressure in 6 months. D Monitor blood pressure next visit, and if > 140/90, initiate antihypertensive therapy. E Her blood pressure indicates prehypertension, there is no need for treatment other than exercise at this time.

B Initiate antihypertensive therapy to target BP < 130/80. The correct answer is (B). According to the JNC-7 guidelines, a patient with diabetes mellitus (DM) or chronic kidney disease (CKD) with this patient's blood pressures should be treated to achieve a goal BP of < 130/80. Although she would be classified as prehypertensive, due to her compelling indication of DM initiation of antihypertensive treatment should begin because she is at higher risk for cardiovascular disease. Lifestyle modifications such as increased exercise, low salt diet, and weight loss are recommended, but in addition to drug therapy at this time due to her increased risk of cardiovascular disease.

A 29-year-old woman presents in July to your office with symptoms of palpitations, sore neck, and excessive sweating, despite using her air conditioner all the time. No surgical or trauma history is noted. She is currently not taking any medications. Vitals include the following: BP = 124/68, pulse = 110 beats per minute, respirations = 18 per minute, and temperature = 101 o F orally. Upon exam, her thyroid is mildly enlarged without nodules, and severely tender. No local erythema or heat is noted. Which of the following lab results would you expect in this patient? A Serum total T4 level = 5.0 ug/dL B Serum TSH level = 0.25 uIU/mL C Sedimentation rate = 15 mm/hr D Free thyroxine index = 8.0 E Positive thyroid stimulating antibodies

B Serum TSH level = 0.25 uIU/mL The correct choice is B, Serum TSH level=0.25 uIU/mL. The reference range for TSH is 0.34 to 4.25 uIU/mL, and therefore the level in this patient is low. This patient is presenting with signs and symptoms of hyperthyroidism, most likely due to subacute thyroiditis. The leaking of thyroid hormone into the circulation causes anterior pituitary suppression and reduced TSH secretion. Choice A is seen in patients with hypothyroidism. Choice C is within the reference range for woman. Since subacute thyroiditis is an acute inflammatory disorder, patients with this disorder will commonly present with an elevated sedimentation rate. Choice D corresponds to a euthyroid situation. It is an estimate of the free thyroid hormone level in the plasma. This result is within the reference range. Choice E is not seen in subacute thyroiditis. They are commonly found in patients with Graves' disease.

A 45-year-old patient presents to your office with a magazine ad for lipitor. She is wondering if she should be taking this medication. She heard on television that it may help to reduce her risk of death. Most of her friends are taking it, and she wants the best health care possible. She has no past medical history or family history of obesity, diabetes, hypertension, stroke, or heart attacks. She doesn't smoke and she exercises regularly. Her diet includes a wide variety of vegetables, fish, meats, and fruit. Her fasting lipid panel reveals the following results: Total cholesterol = 180 mg/dL Total triglycerides = 100 mg/dL HDL = 60mg/dL LDL = 140 mg/dL According to the 2004 revised NCEP ATP III guidelines, what plan should you recommend for this patient? A Initiate strict dietary changes to lower total fat intake to between 10% and 20% daily B Suggest preparing a food diary to look for patterns and choices C Increase the amount of exercise to five days per week D Start lipid lowering agent such as niacin E Suggest starting a low carbohydrate diet

B Suggest preparing a food diary to look for patterns and choices The correct choice is B, suggest preparing a food diary to look for patterns and choices. According to the NCEP ATP III guidelines, this patient does not have any risk factors for coronary heart disease. Her LDL level should be less than 160 mg/dL. All patients can benefit from observing their dietary habits and looking for opportunities to make changes to lower their fat intake. Dietary cholesterol should be less than 200 mg/day, and soluble fiber should be included in the diet. Patients can look at their eating patterns at home, look at the methods used for cooking and baking, and determine the frequency of eating out. Strict changes in her diet, choice A, are not necessary at this time. She already exercises regularly and her HDL is at an adequate level. Therefore, choice C, increasing the amount of exercise to five days per week, is not necessary. Choice D, start a lipid lowering agent such as niacin, is not recommended. Niacin is used to increase serum HDL and lower serum LDL. Her lipid levels are within the reference ranges. Choice E, suggest starting a low carbohydrate diet, may be associated with short-term weight loss, an increase in HDL, and a decrease in triglycerides. This is not something that this patient requires.

At a 2-month-old well-child checkup, a female infant is noted to have the following physical findings: widely open anterior and posterior fontanels, large protruding tongue, coarse facial features, low-set hair line, and an umbilical hernia. In the newborn period, there was a prolongation of physiologic icterus. The results of the newborn screening test are abnormal. Which of the following is the MOST likely diagnosis? A congenital adrenal hyperplasia B congenital hypothyroidism C Crigler-Najjar syndrome D galactosemia

B congenital hypothyroidism Congenital hypothyroidism is one of the most common disorders tested for in newborn screening tests, revealing an elevated TSH (thyroid stimulating hormone) and a decreased T 4 (thyroxine). Symptoms suggestive of congenital hypothyroidism in the neonate include hypotonia, coarse facial features, hirsute forehead, large fontanels (anterior and posterior), widely open sutures, umbilical hernia, protruding/large tongue, hoarse cry, distended abdomen, and prolonged jaundice. Signs of congenital hypothyroidism include lethargy or hypoactivity, poor feeding, constipation, mottling, and hypothermia. Congenital adrenal hyperplasia (CAH) is not universally screened for in the newborn screening test, as it is included in only 14 of the 50 states. In females with CAH, there may be virilization with abnormalities of the external genitalia varying from mild enlargement of the clitoris to complete fusion of the labioscrotal folds. Signs of adrenal insufficiency (salt loss) may present in the first few days of life. Crigler-Najjar syndrome is not one of the disorders tested for in the standard newborn screening tests. It is an inherited disease producing congenital nonobstructive, nonhemolytic, unconjugated severe hyperbilirubinemia. The physical findings in this infant do not correlate with Crigler-Najjar syndrome. Galactosemia is tested for in the newborn screening test in nearly all 50 states. The infant may have symptoms of cataract, hepatomegaly, and prolonged jaundice. Often, these neonates have Escherichia coli sepsis, leading to death in the first 2 weeks of life if not treated promptly.

A 75-year-old man with type 2 diabetes presents to the emergency department with a 2-day history of confusion and lethargy. On physical exam, notable dehydration, tachycardia, and confused mental state is noted. Serum sodium, potassium, magnesium, and chloride levels are normal. The arterial blood gases are normal and serum ketones are negative. The abnormal laboratory findings are as follows: Table shows high glucose and high osmolality Given this information, what is the most likely diagnosis? A diabetic ketoacidosis B hyperglycemic hyperosmolar state C hypoglycemia D dehydration

B hyperglycemic hyperosmolar state A hyperglycemic hyperosmolar state is characterized by dehydration, significant hyperglycemia, and an elevated serum osmolality with an insignificant or negative ketosis. Because of the lack of ketosis, the patient may present with a gradual onset of symptoms, and it can go unnoticed until the dehydration becomes more severe than in ketoacidosis.

A 63-year-old female 3-day's postoperative thyroidectomy complains of tingling around her mouth and feet, and muscle spasms. You note the following finding on exam. What is the most likely cause of her symptoms and physical examination findings? A hypothyroidism B hypoparathyroidism C hyperkalemia D hyperparathyroidism E hypocalcemia

B hypoparathyroidism The correct answer is (B). The patient's recent history of thyroidectomy, together with her symptoms, are suggestive of hypocalcemia due to hypoparathyroidism. Hypoparathyroidism is most commonly caused by removal of the parathyroids during thyroidectomy. The picture above is known as Chvostek's sign, which is suggestive of hypocalcemia. The other choices listed in this question would not cause Chvostek's sign. Hyperparathyroidism causes hypercalcemia.

According to the ATP III revised guidelines, which of the following should be your primary focus of treatment in patients with dyslipidemia? A lowering apolipoprotein B B lowering LDL cholesterol C Lowering non-HDL cholesterol D lowering triglycerides E raising HDL cholesterol

B lowering LDL cholesterol Given their proven efficacy, ease of administration, and enhanced patient compliance over other classes of medications, statin agents are the drugs of first choice for most patients. In particular, patients with diabetes or those in the highest risk category derive special benefits from their use due to their innate anti-inflammatory effects. Myopathy and elevated liver enzymes are the main potential side effects from statin agents. An increase of serum aminotransferase levels to more than three times normal occurs in 1% of patients taking high doses of statins. Monitoring of liver function tests at six weeks, 12 weeks, six months, and annually thereafter can help identify patients with hepatic side effects and facilitate prompt discontinuation of the agents. Rhabdomyolysis occurs in less than 0.1% of cases. It can be prevented by the prompt discontinuation of the agent when muscle pain and elevated muscle enzymes occur. Unexplained pain in large muscle groups should prompt investigation for myopathy; however, routine monitoring of muscle enzymes is not supported by any evidence. Side effects from statins may not be class specific. Therefore, a side effect with one agent should not prevent a trial with another statin agent. Prior concerns about statins causing cataracts or cancer have been alleviated by the release of two large meta-analyses in 2001.

A 78-year-old male returns to the FP office for a follow up of non-insulin-dependent diabetes mellitus (NIDDM) as a new patient to you, although he has been an office patient for the past year. He denies any problems this visit and says his blood sugars are in the 90-120 mg/dl range. He is currently taking the medications listed in the following choices. You receive his labs and note that his creatinine is 2.0 mg/dl and on the previous few labs this creatinine was also in the 1.8-2.0 mg/dl range. What medication should be discontinued? A glipizide B metformin C omeprazole D sitagliptin E atenolol

B metformin The correct answer is (B). Metformin is contraindicated in this diabetic patient with chronic renal failure due to an increased risk of lactic acidosis and should be discontinued. Sitagliptin requires a dosing adjustment in renal failure but is not contraindicated in this patient. Glipizide (sulfonylurea), omeprazole (a PPI for GERD), and atenolol (a beta blocker for hypertension) are not contraindicated in this patient. Beta blockers should be used with caution in diabetics due to the potential of masking symptoms of hypoglycemia, but are not contraindicated.

A 41-year-old woman presents with complaints of weight gain, infrequent menses, and mood changes. You observe her to have moon facies, centripetal fat distribution, and purple striae on her abdomen (see Figure 4-3). Her blood pressure is 152/98 mm Hg. What is the first step in confirming this diagnosis? A random cortisol level B overnight dexamethasone suppression test C thyroid studies D MRI

B overnight dexamethasone suppression test This patient's signs and symptoms indicate possible Cushing syndrome. Overnight dexamethasone testing is the most widely used test, with normal results excluding Cushing syndrome. Cortisol levels are not useful because of diurnal variations.

In addition to insulin and fluid replacement with 0.9% saline, which electrolyte is commonly infused in the type 2 diabetic patient who arrives in the emergency department in a hyperglycemic, hyperosmolar, nonketotic state? A bicarbonate B potassium C calcium D magnesium E sulfate

B potassium Insulin not only causes cellular uptake of glucose but also of potassium. Hypokalemia may develop when insulin is infused to correct either a hyperglycemic hyperosmolar state or a diabetic ketoacidosis. Hence, in order to avoid hypokalemia, potassium chloride can be added to a saline solution, as long as the serum potassium is not elevated.

A 55-year-old man patient presents with tachycardia and heart palpitations. Physical exam shows a multinodular goiter. He does not have obstructive symptoms. He has suppressed TSH and elevated T 3 and T 4 , and a thyroid scan shows multiple functioning nodules. What is the treatment of choice for this patient? A propylthiouracil B beta-blockers C 131I ablation D surgical resection

C 131I ablation The treatment of choice for multinodular goiter is 131 I ablation. In patients with very large thyroid glands with obstructive symptoms, surgical resection may be the best option.

A 45-year-old male comes into your family practice office for his second follow-up appointment since being diagnosed with type 2 diabetes. He does not complain of any symptoms. He is currently taking metformin 1000 mg BID and his HgA 1c at this visit 7.6%. You ask him about his diet and if he is regularly monitoring his blood glucose. He says that he does not really watch what he eats, but he does check his glucose levels daily. He tells you that on average his fasting plasma glucose (FPG) is usually around 88 mg/dl and his postprandial glucose (PPG) is around 180 mg/dl. What would be the BEST next appropriate step in management for this patient? A Nothing, his levels are within normal limits. B Consider switching to basal insulin therapy to control his FPG. C Add a dipeptidyl peptidase 4 (DPP-4) inhibitor to control his PPG. D Add a sulfonylurea to control his PPG. E Increase his metformin to 1500 mg twice daily.

C Add a dipeptidyl peptidase 4 (DPP-4) inhibitor to control his PPG. DPP-4 inhibitors modulated glucagon-like peptide-1 (GLP-1). Their mechanism of action is thought to result from increased incretin levels, especially GLP-1. GLP-1 inhibits glucagon release which, in turn, results in increased insulin secretion, delays gastric emptying, and decreases serum glucose levels. The class is particularly appropriate to utilize in patients who have near-normal HbA 1C and elevated postprandial serum glucose because they work only when food enters the gut and have little to any chance of hypoglycemia.

Which of the following cardiac medications is known to cause clinically significant hypothyroidism? A Furosemide B Captopril C Amiodarone D Digoxin E Dopamine

C Amiodarone The correct choice is C; Amiodarone is an antiarrythmic medication used to treat patients with recurrent ventricular tachycardia or fibrillation. Its structure is similar to thyroid hormone, and its metabolites antagonize thyroid hormone function in approximately 13% of patients treated with amiodrone in the United States. Choice A, furosemide, choice B, captopril, choice D, digoxin, and choice E, dopamine, have no effect on thyroid function.

A new patient is seen in your internal medicine office today. She is coming in to request the removal of several skin tags. She is a 55-year-old woman with a history of untreated acromegaly. A health maintenance plan is set up with the patient, and includes a colonoscopy. This patient is at increased risk for which of the following findings on colonoscopy? A Anal fissures B Ulcerative colitis C Colon polyps D Pseudomembranous colitis E Colonic fistulas

C Colon polyps The correct choice is C, colon polyps. Approximately 30% of patients with acromegaly have been found to have colon polyps. These patients also have an increased risk of colon cancer. Patients with acromegaly have not been found to be at increased risk for the other response choices listed here.

An otherwise healthy 48-year-old male patient presents to your family practice clinic for a complete physical examination. He takes no medications. His vital signs, CBC, CMP, and TSH are normal. His fasting lipid panel reveals a total cholesterol of 280 mg/dL, LDL-C of 190 mg/dL. HDL-C of 38 mg/dL, and triglycerides of 151 mg/dL. What is your next step in his management? A Counsel him on target life style changes and recheck his lipid panel in three months and begin therapy if not to goal at that time. B Redraw his fasting lipid panel today, counsel him on target life style changes and begin therapy immediately if his follow up lipid panel is not to goal. C Counsel him on target life style changes, begin therapy with a statin at this time, and recheck his lipid panel in three months and adjust therapy. D Counsel him on target life style changes, begin therapy with a nicotinic acid derivative at this time, and recheck his lipid panel in three months and adjust therapy. E Counsel him on target life style changes, begin therapy with a fenofibrate at this time, and recheck his lipid panel in three months and adjust therapy

C Counsel him on target life style changes, begin therapy with a statin at this time, and recheck his lipid panel in three months and adjust therapy. The National Cholesterol Education Program (NCEP) Adult Treatment Panel III (ATP-III) and subsequent revisions generally recommends that healthy male patients reach the following goals for cholesterol: low-density lipoprotein (LDL) of less than 100 mg/dL, high-density lipoprotein (HDL) of greater than 40 mg/dl in men and greater than 45 mg/dL in women, and triglycerides of less than 150 mg/dL. While counseling this patient on lifestyle changes is also important, it would be almost impossible for this patient to reduce his levels to goal without medication. A better strategy would be to combine lifestyle changes and initiate medication concurrently. Statins remain the first-line therapy for treatment of dyslipidemias.

A patient presents with symptoms of palpitations, tremor, confusion, and sweating. She is also hungry. Her plasma glucose level is currently 54 mg/dL. Which of the following drugs would be particularly important to investigate during the patient history? A Vancomycin B Acetaminophen C Ethanol D Lisinopril E Prednisone

C Ethanol The correct choice is C, ethanol. Ethanol can suppress gluconeogenesis and stimulate glucose utilization. Alcohol induced hypoglycemia occurs most commonly after a several day drinking binge with little food intake. Other medications that can be associated with hypoglycemia include insulin, insulin secretagogues, salicylates, sulfonamides, pentamidine, quinine, quinolone antibiotics, and beta 1 -adrenergic antagonists. The other choices are not associated with inducing hypoglycemia. Choice E, prednisone use, has been associated with hyperglycemia.

A 36-year-old female patient is being treated for thyroid disease and returns to your family practice office for a six month exam. She is on 100 ugm of levothyroxine once daily. Her vital signs are a blood pressure of 128/82 mm&thinsp;Hg, pulse 62 bpm, respirations 12 bpm, and she is afebrile. Her physical examination is normal. Her thyroid-stimulating hormone (TSH) level is 11.5 uIU/mL (normal 0.4 to 5.0 uIU/mL). What is your next step in her treatment? A Draw a free T3 and free T4. B Lower her levothyroxine to 88 ugm daily and recheck her TSH in 4 to 6 weeks to adjust if necessary. C Increase her levothyroxine to 112 ugm daily and recheck her TSH in 4 to 6 weeks to adjust if necessary. D She is asymptomatic so do not adjust her levothyroxine, but recheck her level at her next visit in six months.

C Increase her levothyroxine to 112 ugm daily and recheck her TSH in 4 to 6 weeks to adjust if necessary. The thyroid gland operates on a reverse feedback mechanism. When TSH is high, free T 3 /T 4 is low and need repletion, and vice versa. In this case, the patient has an elevated TSH, indicating that her serum T 3 /T 4 is likely low and she should have her dose of levothyroxine adjusted upwards and levels can be rechecked starting in approximately four weeks and adjusted subsequently as necessary.

A 4-year-old child is being worked up for type 1 diabetes mellitus. In addition to ordering a fasting plasma glucose and hemoglobin A1c, you would like to determine the presence or absence of any immunologic markers. Which of the following autoantibodies are common in patients with type 1 diabetes mellitus? A Anti-double stranded DNA antibodies B Antinuclear antibodies C Islet beta cell autoantibodies D Pancreatic polypeptide producing cell antibodies E Thyroid stimulating antibodies

C Islet beta cell autoantibodies The correct choice is C, islet beta cell autoantibodies. Islet cell autoantibodies are seen in greater than 75% of patients with type 1 diabetes. These include several different antibodies and serve as a marker of the autoimmune process in the disease. Choice A, anti-double stranded DNA antibodies, is a marker for certain connective tissue disorders, such as systemic lupus erythematosis. Choice B, antinuclear antibodies, are used as a screening test for many different connective tissue disorders, such as systemic lupus erythematosis and scleroderma. Choice D, pancreatic polypeptide producing cell antibodies, are similar to beta cells, but are not involved in the autoimmune process. Choice E, thyroid stimulating antibodies, are found in patients with hyperthyroidism caused by Graves' disease.

A 60-year-old female presents for a routine physical exam. She has not seen a health care provider in over 30 years. She has no significant past medical history and is taking no medications. Her surgical history includes only carpal tunnel repair five years ago. Upon review of symptoms, she notes feeling somewhat tired. She attributes that to old age. Vital signs include BP 110/55, pulse 55 bpm, and respirations 16 per minute. Upon exam you note an enlarged, firm thyroid, thickening of her skin, and puffy facial features and pallor. A portion of blood tests that you ordered is shown below: Serum TSH = 4.4 uIU/mL Hemoglobin = 10.0 g/dL Hematocrit = 30% MCV = 101 Fasting plasma glucose = 105 mg/dL BUN = 10 mg/dL Creatinie = 0.6 ng/mL Which of the following is the most appropriate intervention? A Lithium carbonate 300 mg PO twice daily B Resection of the anterior pituitary C Levothyroxin 50 to 100 ug PO daily D Ferrous sulfate 325 mg PO three times daily E Radioactive iodine protocol

C Levothyroxin 50 to 100 ug PO daily The correct choice is C, levothyroxin 50 to 100 ug PO daily. This patient has classic signs and symptoms of hypothyroidism and required thyroid hormone supplementation. The most common form of hypothyroidism is primary hypothyroidism (e.g. Hashimoto's thyroiditis), and the most common thyroid hormone supplementation is levothyroxine. Choice A, lithium carbonate 300 mg PO twice daily, is a medication used in patients with psychiatric disorders and is known to cause hypothyroidism. Choice B, resection of the anterior pituitary, is not indicated without evidence of a tumor or other pituitary pathology. Secondary hypothyroidism related to the anterior pituitary is quite rare. Choice D, ferrous sulfate 325 mg PO three times daily, is a common treatment protocol for patients with iron deficiency anemia. This patient's MCV is elevated, indicating large red blood cells, as seen in disorders such as vitamin B12 deficiency or folic acid deficiency. Patients with iron deficiency anemia present with microcytic hypochromic anemia. Choice E, radioactive iodine protocol, is a treatment used in patients with hyperthyroidism.

Which of the following sets of disorders is commonly found in multiple endocrine neoplasia (MEN) 2A? A Islet cell tumor; renal cell carcinoma; pheochromocytoma; B Pheochromocytoma; medullary thyroid carcinoma; mucosal neuromas C Medullary thyroid carcinoma; parathyroid hyperplasia; pheochromocytoma D Parathyroid adenoma; islet cell hyperplasia; pituitary adenoma E Visceral lipomas; Marfanoid features; retinal angiomas

C Medullary thyroid carcinoma; parathyroid hyperplasia; pheochromocytoma The correct choice is C. The three primary features of MEN type 2A include medullary thyroid carcinoma, parathyroid hyperplasia or adenoma, and pheochromocytoma. In choice A, islet cell tumor and renal cell carcinoma are disorders found in various other MEN syndromes, but not in MEN type 2A. Choice B, pheochromocytoma, medullary thyroid carcinoma, and mucosal neuroma are disorders found in MEN 2B. Choice D, parathyroid adenoma, islet cell hyperplasia, and pituitary adenoma are found in MEN I. Choice E, visceral lipomas, Marfanoid features, and retinal angiomas are features found in various MEN syndromes, but not in MEN 2A.

A 60-year-old female presents to her primary care practitioner for her yearly check up. Her past medical history is significant only for a history of repeat kidney stones. She enjoys gardening outside, has no significant family history, and is not taking any medications. Routine mammograms have been normal. During the review of her systems, the patient describes feeling tired lately and has noticed muscle aches over the last few weeks. Initial lab values are shown below: Hemoglobin = 12.0 gm/dL Hematocrit = 36% BUN = 10 mg/dL Creatinine = 0.7 ng/mL Calcium = 12 mg/dL Phosphate = 2.0 mg/dL PTH = 100 pg/mL TSH = 2.0 UIU/mL What is the most likely cause of her symptoms? A Vitamin D deficiency B Hypercalcemia of malignancy C Primary hyperparathyroidism D Factitious hypercalcemia E Hyperthyroidism

C Primary hyperparathyroidism The correct choice is C, primary hyperparathyroidism. This patient has the characteristic signs and symptoms of hypercalcemia, along with the lab findings of primary hyperparathyroidism with elevated calcium, low phosphate, and elevated PTH. Other common presenting features include bone pain, paresthesias, polyuria, and gastrointestinal symptoms. Females are affected three times as much as males, and it is typically found accidentally. Choice A, vitamin D deficiency, is unlikely since she enjoys gardening outside in the sun. Vitamin D deficiency can be seen is association with primary hyperparathyroidism, but is not the cause of this disorder. Choice B, hypercalcemia of malignancy, is unlikely with her history of normal mammograms. Serum PTH is typically normal in this disorder as well. Choice D, factitious hypercalcemia, is unlikely since she doesn't take any calcium supplements or other medications. Choice E, hyperthyroidism, can be a cause of hypercalcemia if the patient is suffering from thyrotoxicosis, but this patient's TSH is within the reference range.

What is the definitive treatment for the majority of patients presenting with mild symptoms of hyperthyroidism secondary to subacute thyroiditis? A Subtotal thyroidectomy B Oral methimazole C Symptomatic treatment D Radioactive iodine E Antibiotics

C Symptomatic treatment The correct choice is C, symptomatic treatment. Most patients with subacute thyroiditis and symptoms of hyperthyroidism require only symptomatic treatment, with non-steroidal anti-inflammatory medications and/or beta blockers, for any cardiac symptoms including palpitations and tachycardia. Occasionally, patients may require a course of prednisone for this acute inflammatory condition. Most patients will recover spontaneously within a few months. Choices A, B, D, and E are not necessary in this condition. Most cases of subacute thyroiditis are associated with viral infections, and resolve without additional thyroid medications.

A patient presents with symptoms of polyuria for several months. There is no history of diabetes mellitus or intrinsic kidney disease. Which of the following tests can best determine whether the polyuria is resulting from primary polydipsia or some form of diabetes insipidus? A 24-hour urine volume test B Routine urinalysis C Water deprivation test D Urine culture and sensitivity E Vasopression suppression test

C Water deprivation test The correct choice is C, water deprivation test. This test helps the practitioner determine if the patient can concentrate his/her urine with or without becoming hyperosmolar. The patient is restricted from liquids and food during the test, which must be done in a controlled environment. A patient with primary polydipsia will be able to concentrate his/her urine without becoming hyperosmolar. A patient with diabetes insipidus will become hyperosmolar without concentrating the urine. Choice A, 24-hour urine volume test, can provide quantification of the polyuria, but will not determine its cause. Choice B, routine urinalysis, may provide evidence of dilute urine, but will not provide evidence of its cause. Choice D, urine culture and sensitivity, is used when trying to determine the cause of a urinary tract infection. Choice E, vasopression suppression test, is not a test that is performed. It could be dangerous to try to suppress a hormone that is already thought to be deficient.

Your patient states she has been gaining weight for no apparent reason over the past year and is concerned that she might have hypothyroidism. What other historical information would support a diagnosis of hypothyroidism? A anxiety B diarrhea C depression D palpitations E heat intolerance

C depression The correct answer is (C). Depression is a common presenting symptom of hypothyroidism. Weight gain can occur with hypothyroidism. Other symptoms may be weakness, fatigue, and menorrhagia. Hoarseness may also be a presenting symptom. The other choices are common symptoms of hyperthyroidism.

42-year-old female complains of weight gain (especially in her abdomen) over the past 8 months. She also has noted that her skin bruises easily. Her husband has noted she seems to be very moody lately and she is worried about their relationship. Furthermore, her hair seems to be getting thinner and she is now getting acne like she had in her teenage years. She wonders if this is due to menopause since her periods have stopped suddenly about a year ago. On physical examination her BP = 170/50, P = 82, T = 98.2˚F. You note the following findings on examination (see picture). What diagnostic test is indicated initially to confirm your suspected diagnosis? Photo shows purple striae A cosyntropin stimulation test B MRI pituitary C dexamethasone suppression test D 24-hour urine for protein E serum protein electrophoresis

C dexamethasone suppression test The correct answer is (C). The patient's symptoms are consistent with a diagnosis of Cushing's syndrome (or disease). Her physical examination findings of hypertension and abdominal obesity with the classic purple striae also support the diagnosis. The initial diagnostic test of choice would be the dexamethasone suppression test. If the test is positive, further confirmatory testing is done which would also help to identify the cause. An MRI of the pituitary is appropriate if further testing suggests the possibility of a pituitary adenoma as the cause of the Cushing's syndrome, but is not used as an initial diagnostic test for Cushing's disease. A cosyntropin stimulation test, choice (A), is indicated for the diagnosis of Addison's disease.

A 41-year-old female comes in for her annual exam. She generally feels well and has no complaints other than some general fatigue. On examination you note a normal size thyroid with a palpable firm 1.5 cm nodule in the left lobe of her thyroid. What is the best test to order that will provide a definitive diagnosis? A thyroid ultrasound B thyroid uptake scan C fine needle biopsy of nodule D MRI thyroid E TSH

C fine needle biopsy of nodule The correct answer is (C). The patient has a solitary, > 1cm, firm nodule, which is suspicious for a thyroid malignancy. A fine needle biopsy of the nodule would provide you with a definitive diagnosis. A thyroid ultrasound, thyroid uptake scan, and TSH would be helpful in determining the etiology of the nodule but will not provide you with a definitive diagnosis. Since this patient has a suspicious clinical examination a biopsy is indicated. MRI of the thyroid is not a preferred test.

Your supervising physician asks you to advise him which finding is least likely to be suggestive of a thyroid malignancy in your 49-year-old female with a small palpable thyroid nodule. Which of the following choices would be least likely to suggest malignancy in this patient? A ultrasound showing lesion with microcalcifications B ultrasound showing a lesion of > 1 cm C hot nodule on 123I uptake scan D ultrasound showing a solid lesion E cold nodule on 123I uptake scan

C hot nodule on 123I uptake scan The correct answer is (C). A hot nodule, which is a hyperfunctioning thyroid nodule, suggests a benign etiology. The other choices, including ultrasound findings of microcalcifications, solid lesions, and lesions > 1 cm, should increase your index of suspicion for possible malignancy. Cold nodules are nonfunctioning thyroid nodules, which should increase your suspicion, especially in combination with suspicious ultrasound and/or clinical examination findings.

A 23-year-old woman, adopted in infancy, has just learned that her biologic family has a long history of tumors of the thyroid and parathyroid glands, the pancreas, the adrenal glands, and of lipomas. Given this history, she undergoes genetic testing and is found to have a mutation in the menin gene on the long arm of chromosome 11 (11q13). What is the most likely initial clinical manifestation for persons with this syndrome? A gastrinoma B Hirschsprung disease C hyperparathyroidism D medullary thyroid cancer E pituitary adenoma

C hyperparathyroidism This genetic mutation is found in patients with MEN1 syndrome, formerly known as Wermer Syndrome. About two-thirds of patients with this syndrome present with hyperparathyroidism. Gastrinomas (A) occur in about one-third and pituitary adenomas (E) in about half. Hirschsprung disease (B) and medullary thyroid carcinomas (D) are found in MEN2a syndrome.

A 30-year-old patient presents 2 months postthyroidectomy. The patient has had symptoms of increased irritability, muscle spasms, and hair loss for the past month. On physical examination, a positive Chovstek sign is noted. Which of the following is the most likely diagnosis? A hypothyroidism B hypopituitarism C hypoparathyroidism D hypogonadism

C hypoparathyroidism Hypoparathyroidism commonly presents following thyroidectomy surgery. This patient has classic signs and symptoms of a low calcium level and hypoparathyroidism. Chovestek sign is a physical exam finding that is positive after tapping in front of the ear in the facial nerve region. When doing this, the muscle contracts. When the calcium level is low, this occurs. Hypothyroidism can occur following a thyroidectomy but the symptoms are not the same.

Your patient has a history of primary hyperparathyroidism. Recently she has been hospitalized due to obstructing kidney stones. She has had several fractures including her hip, sacrum, and forearm in the past year, all on separate occasions. She is constantly complaining of a lack of energy. What is the recommended treatment for her symptomatic hyperparathyroidism? A surgical removal of the pituitary B high-dose calcium supplementation C parathyroidectomy D thyroidectomy E thiazide diuretics

C parathyroidectomy The correct answer is (C). Primary hyperparathyroidism is most commonly secondary to a single parathyroid adenoma. The recommended treatment for symptomatic primary hyperparathyroidism is parathyroidectomy. High-dose calcium supplementation and thiazide diuretics, choices (B) and (E), can worsen the hypercalcemia associated with hyperparathyroidism. Neither surgical removal of the pituitary, choice (A), nor or a thyroidectomy, choice (D), is an indicated treatment for this condition.

Which of the following medications for type 2 diabetes mellitus is thought to best preserve beta cell function in the pancreas? A acarbose B glimepiride C pioglitazone D sitagliptin E insulin glargine

C pioglitazone Pioglitazone is a thiazolidinedione (TZD). TZDs are thought to help preserve beta-cell function.

You are treating a 60-year-old man with a history of angina. He has been on the therapeutic lifestyle change (TLC) diet for 12 weeks (with solid effort). This patient has no other medical conditions and takes nitroglycerin as needed and daily enteric-coated aspirin. His fasting lipid panel from last week demonstrates the following: total cholesterol- 295 mg/dL LDL-145mg/dL HDL-48mg/dL What is the most appropriate treatment at this time? A prescribe colestipol B prescribe ezetimibe C prescribe simvastatin D prescribe niacin E no pharmacological treatment

C prescribe simvastatin This patient's coronary heart disease risk factors and failed TLC diet warrant pharmacological treatment based on his LDL level. Although there are no absolute guidelines for the selection of lipid-modifying medications, an HMG-CoA reductase inhibitor is preferred.

Over a period of several months, a 62-year-old man has developed erectile dysfunction. He has no history of neurologic, kidney, or cardiovascular disease or diabetes mellitus. He takes a multivitamin and an occasional ibuprofen for aches and pains. He has never smoked cigarettes, drinks 1-2 glasses of wine with dinner on weekends, and uses no mind-altering drugs. Physical examination is remarkable only for bilateral gynecomastia. What is the most likely diagnosis? A breast cancer B depression C prolactinoma D steroid abuse E testicular cancer

C prolactinoma Men with prolactinomas may experience erectile dysfunction, infertility, and, less commonly, gynecomastia. Breast cancer in men (A) presents as a usually as a unilateral mass. Men with depression (B) may have erectile dysfunction, but not gynecomastia. Steroid abuse (D) is associated with gynecomastia, but the patient would likely be showing other signs and symptoms. Testicular cancer (E), specifically germ cell cancer, is associated with gynecomastia in 5% of cases but this man has no testicular mass or swelling.

A 50-year-old male is seen with a routine check-up. He is concerned about the possibility of developing diabetes mellitus. He has a negative family history of diabetes. He has no signs or symptoms of diabetes and he is not overweight. Without any risk factors for diabetes, what is the recommended screening protocol for this patient according to the American Diabetes Association (ADA)? A screen all men over 25 years of age every five years B screen all men over 35 years of age every two years C screen everyone over 45 years of age every three years D no screening is necessary without risk factors E no screening is necessary without a family history of diabetes

C screen everyone over 45 years of age every three years The correct choice is C, screen everyone older than 45 years of age every three years. In addition, the ADA recommends screening for younger people if they are overweight and have at least one additional risk factor, such as positive family history, hypertension, and/or vascular disease. The other choices are not recommended by the ADA for screening the general population for diabetes mellitus.

A 43-year-old obese man presents for a health maintenance visit. On physical exam, it is noted that his waist circumference is 106 cm and blood pressure is 148/92 mm Hg. Which of the following fasting laboratory levels would suggest a diagnosis of metabolic syndrome (syndrome X) in this patient? A HDL of 45 mg/dL B LDL of 180 mg/dL C triglyceride of 190 mg/dL D glucose of 100 mg/dL

C triglyceride of 190 mg/dL Metabolic syndrome is found in approximately 25% of Americans. It is defined as three or more of the following findings: waist circumference of greater than 102 cm in men or greater than 88 cm in women; serum triglyceride level of at least 150 mg/dL, HDL level of less than 40 mg/dL in men or less than 50 mg/dL in women; blood pressure of at least 130/85 mm Hg; and serum glucose level of at least 110 mg/dL.

A 70-year-old woman who was found barely responsive at home by her daughter is brought to the emergency department. Evaluation reveals that she is in a hyperglycemic hyperosmolar state with a severe fluid deficit. Treatment is initiated with vigorous saline rehydration and a continuous infusion of insulin. At what point should her glucose be added to her treatment? A when her condition becomes stable B when her urine output reaches 50 mL/hour C when her blood glucose reaches 250 mg/dL D if she develops hypokalemia E if she begins to spill ketones in her urine

C when her blood glucose reaches 250 mg/dL In hyperglycemic hyperosmolar states, the serum glucose rapidly corrects with fluid administration alone. However, with vigorous rehydration, glucose may fall precipitously and lead to severe hypoglycemia. To avoid this, glucose should be added to water, half-normal, or normal saline as soon as the patient's blood glucose is less than or equal to 250mg/dL. She should continue to receive insulin IV until she is stabilized (A) when it can be switched to subcutaneous administration. The goal of fluid therapy in this patient is restoring her urine output to 50 mL per hour (B) or more. Because insulin drives potassium into the cells and can cause hypokalemia (D), potassium chloride should be given unless the patient has chronic kidney disease or oliguria. Persons in a hyperglycemic hyperosmolar state typically do not spill ketones (E) the way persons with diabetic ketoacidosis do.

One of your patients is requesting your help. He has seen three physicians in the past 3 months and is still having symptoms. He states, "I keep having these episodes of feeling like I am going to die. Out of the blue I feel real nervous, I get a splitting headache, break out in a sweat, and even feel like I am trembling. Sometimes I feel my heart beating out of my chest but don't have any pain or shortness of breath." He has had numerous tests including a cardiac stress test, multiple EKGs, complete blood count (CBC), basic metabolic panel (BMP), thyroid tests, and computed tomography (CT) scans of his head, which were normal. His last physician told him that he had an anxiety disorder and should try medications; he declined. His only medical problem is hypertension, which has worsened recently. He is currently taking lisinopril 20 mg QD, amlodipine 10 mg QD, and HCTZ 25mg QD. His physical examination is unremarkable except a BP of 190/92, P = 74. What is the best test to order to confirm your suspected diagnosis? A dexamethasone suppression test B sleep study C renal artery ultrasound D plasma fractionated free metanephrines E thyroid uptake scan

D plasma fractionated free metanephrines The correct answer is (D). The patient's symptoms and uncontrolled hypertension with a previous negative evaluation for cardiac, electrolyte, or thyroid causes highly suggests a pheochromocytoma as the possible cause. Plasma fractionated free metanephrines is a very sensitive test for the diagnosis of pheochromocytoma. Another test to consider would be evaluation of the total urinary metanephrines. A dexamethasone suppression test, choice (A), is used to r/o Cushing's syndrome, which is unlikely due to the normal physical examination findings. A sleep study, choice (B), (suggesting sleep apnea) and renal artery ultrasound, choice (C), (suggesting renal artery stenosis) are used to diagnose secondary hypertension, but would unlikely explain all the symptoms in the scenario. A thyroid uptake scan, choice (E), is not indicated since there is no indication of abnormal thyroid testing suggesting hyperthyroidism, or examination stated findings suggestive of a thyroid nodule.

You are examining n 42-year-old male with the following physical examination findings (see picture below). What symptoms would most likely correspond to his diagnosis? Photo is of acanthosis nigricans A anxiety B palpitations C excessive snoring D polyuria and polydipsia E weight gain

D polyuria and polydipsia The correct answer is (D). The patient's physical examination findings suggest acanthosis nigricans, a condition associated with diabetes mellitus. This patient would most likely present with polydipsia and polyuria at diagnosis. He may have weight loss and fatigue. Anxiety and palpitaions are not as likely to present symptoms of diabetes.

A 12-year-old boy is being seen for concerns of development of breast tissue. Upon physical exam, he is noted to have a firm, slightly tender mass under the left areola. What is the most appropriate action at this time? A referral to pediatric surgery for resection B measurement of serum hCG C measurement of testosterone and estrogen levels D reassurance and observation

D reassurance and observation Type 1 idiopathic gynecomastia in adolescent men presents with a firm mass under the areola ("breast bud") typically during sexual maturation stages (SMR), stages II to III. This is a result of normal estrogen and androgen activity at the breast tissue level. Appropriate action is observation and to reassure the patient that the condition will likely resolve in 1 to 2 years

A young man presents with difficulty breathing at times. Upon exam you note evidence of a firm, fixed thyroid nodule with extension toward the trachea and surrounding muscles. The patient has a family history of thyroid cancer. You are concerned that the patient may have medullary thyroid cancer. After you confirm your suspicion, which of the following genetic tests would you suggest to his family members? A BRCA 1 B APC C CFTR D ret proto-oncogene E CAG triplet expansion

D ret proto-oncogene The correct choice is D, ret proto-oncogene. Genetic testing to identify this oncogene is available for patients whom you suspect have medullary thyroid cancer and/or pheochromocytoma, and for their asymptomatic families who may be at risk for the same disorders. A mutation in this gene leads to uncontrolled growth of the C cells of the thyroid. Choice A, BRCA 1, is an oncogene that is associated with risk for breast, ovarian, colon, and prostate cancers. Choice B, APC, is an oncogene noted in the familial colon cancer syndrome known as familial adenomatous polyposis. DNA analysis for CFTR mutations, choice C, can identify the gene mutations in patients with cystic fibrosis in the vast majority of instances. Choice E, CAG triplet expansion, is the result of a gene mutation in Huntington's disease.

An extremely heavy 12-year-old girl comes to the practice with her grandmother for new patient evaluation, bringing old records with her. Her blood pressure today is mildly elevated. Which of the following parameters will help determine whether her overweight and elevated blood pressure are due to Cushing syndrome (adrenocortical hyperfunction) rather than exogenous obesity? A advanced skeletal maturity B heavy thighs and legs C pinkish striae D short stature E slightly increased growth rate

D short stature Children with Cushing syndrome typically have short stature, while those who are obese due to exogenous factors have normal or tall stature. Likewise, they tend to have delayed skeletal maturity (A), truncal obesity with thin extremities (B), purplish striae (C), and a slowed growth rate (E), while obese children have advanced maturation, heavy extremities, pinkish striae, and an increased growth rate.

A 53-year-old man is taking a proton pump inhibitor for GERD symptoms, a beta blocker and a thiazide diuretic for hypertension, an SSRI for depression, and an over-the-counter NSAID as needed for aches and pains. He has developed gynecomastia and laboratory studies reveal an elevated prolactin level. If his hyperprolactinemai is due to one of his medications, which is the most likely cause? A the beta blocker B the NSAID C the proton pump inhibitor D the SSRI E the thiazide diuretic

D the SSRI Many medications cause hyperprolactinemia, including SSRIs, tricyclic antidepressants, and antipsychotics. Hydralazine and methyldopa, but not beta blockers (A), may also raise prolactin levels; likewise opioids, but not NSAIDS (B). Cimetidine and ranitidine, but not proton pump inhibitors (C) are included among possible pharmaceutical causes. Thiazide diuretics (E) are not know to raise prolactin levels.

Following a total thyroidectomy for papillary carcinoma, a 72-year-old man develops a heart rate of 140 and a temperature of 104.8F. He vomits almost continuously and has severe diarrhea. He is disoriented and mildly combative. Electrocardiography demonstrates sinus tachycardia. What is the most likely diagnosis? A pulmonary embolism B pneumonia C sepsis D thyroid storm E wound infection

D thyroid storm Although rare, thyroid storm or crisis can occur following thyroid surgery, administration of radioactive iodine, or a stressful illness. It is characterized by extreme tachycardia, vomiting, diarrhea, dehydration, delirium, and high fever. Pulmonary embolism (A) may include tachycardia, along with chest pain and shortness of breath, but is unlikely to include GI symptoms. Pneumonia (B) and sepsis (C) are certainly possible, although the extreme nature of her findings more strongly suggest thyrotoxicosis. Wound infection (E) would more likely to present with localized tenderness and a less marked fever.

Your patient returns to your office for a follow up for non-insulin-dependent diabetes mellitus (NIDDM). Her HgA1c in the office is 6.4%. She is concerned about developing kidney disease from her diabetes and requests that you test her for this. What initial screening test should you order that would provide clues to potential diabetic nephropathy allowing for treatment to slow the disease progression? A 24-hour urine for protein B serum BUN/CR C urine microscopic D urine microalbumin E serum protein

D urine microalbumin The correct answer is (D). An easy office dipstick or laboratory test for urine microalbumin should be done initially and periodically on diabetic patients who are at risk for diabetic nephropathy. Treatment should be initiated if microalbuminuria is found to slow disease progression. A urine microscopic for renal casts may be helpful if the patient has symptoms of kidney disease, but is not an initial screening test. Serum BUN/CR and GFR are useful tests for patients with known diabetic nephropathy to indicate the stage of chronic renal failure but is not elevated early in the disease progression, before urine microalbumin. A 24-hour protein is not indicated in this case as an initial screening test.

What is the radionuclide imaging pattern noted during a thyroid scan in patients with subacute thyroiditis? A Single area of increased uptake B Diffusely high uptake C Multiple areas of increased uptake D Single area of low uptake E Diffusely low uptake

E Diffusely low uptake The correct choice is E, diffusely low uptake. Acute inflammation is occurring during subacute thyroiditis, causing leakage of stored thyroid hormone into the circulation. The thyroid is not metabolically active, and therefore there is low radioactive iodine uptake on scan. This is in distinction to the diffuse increased uptake seen on scan in Graves' disease, choice B. Choice A, a single area of increased uptake, is commonly seen with a "hot" or toxic nodule. Choice C, multiple areas of increased uptake, are found in patients with multinodular goiters. This patient had no nodules noted on exam. Choice D, single area of low uptake, is seen as cold nodules and may be related to a benign nodule or thyroid cancer.

A 47-year-old man presents with signs and symptoms of Cushing's syndrome. He has no history of depression, alcoholism, anorexia, or medication use. His past medical history is negative for any known malignancies. An MRI of the pituitary gland is not definitive for any masses. You are trying to decide if the symptoms are caused by an occult pituitary adenoma or occult malignancy in another area of the body. Which of the following is the definitive test used to differentiate pituitary from non-pituitary ACTH dependent Cushing's syndrome? A 12:00 a.m. plasma cortisol B Random plasma ACTH C 24-hour urine free cortisol D MRI of the adrenal glands E Inferior petrosal sinus ACTH

E Inferior petrosal sinus ACTH The correct choice is E, inferior petrosal sinus ACTH. This test can definitively indicate whether or not an elevated plasma ACTH level is due to secretion from the pituitary or an ectopic source. Simultaneous measurements of ACTH from the peripheral circulation and the inferior petrosal sinus after CRH stimulation can help to locate the surge's origin. Highly accurate results can be obtained if this test is performed by a skilled interventional radiologist. Although the other choices listed are used in the evaluation of patients with hypercortisolemia, they do not detect any related ACTH surge source.

A patient is recovering from having a total thyroidectomy two days ago for medullary thyroid cancer. An extensive neck dissection was required during the surgery. Post-operative lab testing reveals a low serum calcium level. Which of the following clinical presentations will most likely occur in this patient? A Constipation B Anorexia C Polyuria D Bone pain E Paresthesias

E Paresthesias The correct choice is E, paresthesias. Circumoral paresthesias are signs of hypocalcemia. Hypocalcemia can occur after any type of neck surgery that may have resulted in destruction of the parathyroid glands. Choices A through D are symptoms of hypercalcemia and may be seen in hyperparathyroidism.

A 35-year-old male presents complaining of increasingly constant headaches, double vision centrally, and a progressive loss of peripheral vision for two weeks. He has no previous headache history and denies any other medical conditions. Physical examination reveals bitemporal hemianopsia without additional neurologic findings. What is the most likely diagnosis? A Acute ischemic stroke B Circle of Willis ruptured aneurysm C Migraine headache D Multiple sclerosis E Pituitary adenoma

E Pituitary adenoma Pituitary adenomas, benign neoplasms associated with pituitary hormone secretory changes, may enlarge and become symptomatic. Symptoms are based upon the location and size of the tumor, and may include bitemporal hemianopsia, double vision, color desaturation, and visual acuity loss. Headaches may occur, due to associated pressure changes within the intrasellar space. Additional evaluation should include a T1-weighted MRI, screening laboratory tests, and a full ophthalmologic evaluation. These tests will also help evaluate for potential differential diagnoses, such as those listed. The patient's history is not consistent with an acute ischemic stroke or migraine headache. Although an unruptured aneurysm may have very similar findings to a pituitary tumor, ruptured aneurysms present with acute headache, nausea, vomiting, and potential changes in consciousness. Multiple sclerosis (MS) should remain on the differential for this patient and will also be evaluated through MRI (although the current findings are more consistent with a pituitary adenoma), and additional neurologic findings would be likely with MS.

Which of the following pairs of conditions constitute 90% of the causes of adult hypercalcemia? A Vitamin D intoxication and renal failure B Lithium ingestion and hyperthyroidism C Prolonged immobilization and thiazide use D Hypothyroidism and hypoparathyroidism E Primary hyperparathyroidism and malignancy

E Primary hyperparathyroidism and malignancy The correct choice is E, primary hyperparathyroidism and malignancy. Together, they constitute 90% of the causes of hypercalcemia. Choices A, B, and C can cause hypercalcemia, but at a lower rate. Choice D, hypothyroidism and hypoparathyroidism, do not cause hypercalcemia.

A 45-year-old patient came in to see his health care provider today, to discuss the results of his last annual assessment. He was told that he had developed type 2 diabetes mellitus. One of the recommendations from the physician assistant included a visit to an ophthalmologist. The physician assistant was concerned after seeing new capillaries, macular edema, and fibrous tissue within the retina during his funduscopic exam. What type of ocular complication does this patient most likely have at this time? A Background retinopathy B Closed angle glaucoma C Macular degeneration D Diabetic cataracts E Proliferative retinopathy

E Proliferative retinopathy The correct choice is E, proliferative retinopathy. The distinguishing factor in the patient's presentation, which signals this disorder, is the development of newly formed vessels. Proliferative retinopathy is the leading cause of blindness in the United States. Up to 20% of patients with type 2 diabetes have retinopathy at the time of diagnosis. Choice A, background retinopathy, or simple retinopathy includes retinal microaneurysms, hemorrhages, exudates, and edema, without new vessel formation. Choice B, closed angle glaucoma, is relatively uncommon in patients with diabetes, except after cataract extraction. Choice C, macular degeneration, is not associated with diabetes mellitus specifically. Choice D, diabetic cataracts, tends to occur in patients with diabetes earlier than the general population, and may correlate with the severity of the disease.

A 40-year-old patient presents to your walk-in clinic with symptoms of hyperhydrosis, oily skin, daytime sleepiness, and snoring. Upon exam, you note large fleshy heel pads and hands with sweaty palms. The patient also has coarse facial features. When asked, the patient isn't aware of any major changes in her face or body. She has not seen another health care provider in many years and has not kept up with any health care maintenance schedule. The patient lives with her ill mother and is not currently employed. Which of the following screening tests would best aid in the diagnosis of this patient? A CT of the chest and abdomen B Thyroid scan C 24-hour urine for catecholamines D Serum calcitonin level E Serum IGF-I level

E Serum IGF-I level The correct choice is E, serum IGF-I level. Age and gender matched levels of IGF-I are elevated in patients with acromegaly. IGF-I is the mediator of most of the effects of GH on the body, and lead to the proliferation of bone, cartilage, and soft tissue. Although GH levels may be elevated in patients with acromegaly, they are secreted in a pulsatile fashion and are not consistently elevated. Serum GH levels are not the best screening test for acromegaly. Choices A, a CT of the chest and abdomen, and B, a thyroid scan, are expensive imaging studies that are not usually used as screening tests. They also have no role in the routine workup of patients with suspected acromegaly. Choice C, 24-hour urine for catecholamines, is a test that can be used in the work up of patients with suspected pheochromocytoma. Choice D, serum calcitonin levels, are associated with medullary thyroid cancer and other thyroid disease.

A 29-year-old woman presents in July to your office with symptoms of palpitations, sore neck, and excessive sweating, despite using her air conditioner all the time. No surgical or trauma history is noted. She is currently not taking any medications. Vitals include the following: BP = 124/68, pulse = 110 beats per minute, respirations = 18 per minute, and temperature = 101 o F orally. Upon exam, her thyroid is mildly enlarged without nodules, and severely tender. No local erythema or heat is noted. What is the most likely diagnosis in this patient? A Thyroid abscess B Papillary thyroid cancer C Hashimoto's thyroiditis D Graves' disease E Subacute thyroiditis

E Subacute thyroiditis This patient is presenting with signs and symptoms of hyperthyroidism, most likely caused by subacute thyroiditis. The enlarged and tender thyroid, along with the fever, are classic findings in this condition. Choice A, thyroid abscess, is a localized infection of the thyroid. It would present with heat, swelling, and erythema in the area of tenderness. Choice B, papillary thyroid cancer, commonly presents with either a local non-tender thyroid nodule or as lymph node metastasis. Choice C, Hashimoto's thyroiditis, is a cause of chronic thyroiditis and is associated with a clinical presentation of hypothyroidism. Choice D, Graves' disease, is the most common cause of hyperthyroidism, but does not present with thyroid tenderness and fever.

A 10-year-old child is seen with his parents for a routine check up. During the review of symptoms, his parents mention that their son has been extremely thirsty and is going to the bathroom to urinate frequently. The patient agrees. The parents are concerned that their son has developed diabetes mellitus. The family history is negative for diabetes mellitus, but the mother has a history of familial hypothalamic diabetes insipidus. Screening blood work includes a CBC, hemoglobin A1c, and renal function tests, all of which are within the reference range. Which of the following serum analytes would you expect to be deficient? A Sodium B Glucose C Thyroxine D Prolactin E Vasopressin

E Vasopressin The correct choice is E, vasopressin. The reader must first understand that the patient's symptoms are classic for diabetes insipidus, with the increased thirst, frequency, and polyuria. These symptoms are also seen in diabetes mellitus, but this would be less likely since the patient doesn't have a family history of diabetes mellitus, and his hemoglobin A1c is not elevated. Choice A, sodium, should be within range, as long as the boy is able to drink when thirsty; otherwise, he would become hypernatremic. Choice B, glucose, is incorrect. Although the signs and symptoms also fit the pattern of diabetes mellitus, the glucose would most likely be elevated in this disorder, and not low. Choice C, thyroxine, is under control of TSH from the anterior pituitary gland, and choice D, prolactin, is secreted from the anterior pituitary gland. Vasopression is secreted from the posterior pituitary gland.

A 12-year-old boy who is 60" tall and weighs 190# is found on routine physical examination to have 2+ glucose and trace ketones in his urine. His fasting glucose is 140 mg/dL and hemoglobin A1C is 6.0%. What is the next step in this child's management? A Initiation of insulin treatment B Prescription of oral metformin C Referral for a glucose tolerance test D Watchful waiting E Weight loss and exercise

E Weight loss and exercise Treatment of type 2 diabetes and pre-diabetes in children varies with the severity of the disease. If the HbA1C is near normal and ketones are not significantly increased, the first line of treatment is lifestyle modifications, including nutrition counseling for the entire family, weight loss, and exercise. Insulin (A) is not indicated in early type 2 disease. If life style changes are not successful, addition of metformin (B) is the next step. Glucose tolerance tests (C) are rarely needed in children. Watchful waiting (D) is inappropriate as continued elevated glucose levels put the child at risk for micro- and macrovascular damage.

Which of the following statements regarding diabetic medications is/are most correct? A Incretin-mimetics like exenatide commonly result in weight loss. B Thiazolidinediones (TZDs) should be held prior to and for 48 hours after administration of ionidated contrast material. C Sulfonylureas have the highest risk of hypoglycemia of all oral diabetic agents. D Sulfonylureas help preserve beta-cell function. E All of the above.

E all of the above Incretin-mimetics act upon the satiety center of the brain and as a result promote weight loss, averaging approximately six pounds. Metformin may cause fatal lactic acidosis when given to patients on concomitantly-administered iodinated contrast dye. Sulfanylureas, in part because of first-pass metabolism, have the highest risk of hypoglycemia of all the oral anti-diabetic agents. TZDs have been found to preserve beta-cell function more so than any other agent.

A 33-year-old female with history of non-insulin dependent diabetes mellitus (NIDDM), hypertension, and obesity comes to see you for treatment of her significantly elevated cholesterol. She intends to have children in the next year and has been off birth control. Which of the following medications is considered to be safest for treatment of hyperlipidemia in pregnancy? A lovastatin B ezetimibe C rosuvastatin D fenofibrate E colesevelam

E colesevelam

A 13-year-old girl is brought to the office by her mother who is having difficulty managing the child's recent angry outbursts. Her grades in schools have gone from A's and B's to C's and D's; she has been sent to the office for "behavior problems" several times in the last two months despite having been an exemplary student in the past. The girl says she "feels nervous" all the time, "just can't sit still" in class, and "sweats like a pig" even when she isn't exercising. Her weight today is 10# less than at her previous well-child visit 6 months ago, but the child denies dieting and her mother reports that she eats "constantly." Vital signs include temperature of 99.0F, pulse of 96, respirations of 26. Her blood pressure is 148/82. Her skin is warm and moist. Which of the following is the most likely diagnosis? A amphetamine abuse B anorexia nervosa C bipolar disorder D bulimia nervosa E hyperthyroidism

E hyperthyroidism Hyperthyroidism occurs more common in girls than in boys, most frequently in adolescence. Deterioration in school performance is a common finding, along with emotion lability, nervousness, personality changes, warm and moist skin, polyphagia, and weight loss. Systolic hypertension and a widened pulse pressure are often found. Amphetamine abuse (A) is certainly a possibility, but it tends to depress the appetite. Anorexia nervosa (B) is unlikely given the child's appetite. Bipolar disorder (C) in adolescents more typically presents with depressive symptoms, and does not explain the physical findings. Patients with bulimia nervosa (D) are typically normal or overweight.

A 25-year-old woman comes in for her annual physical examination and renewal of her oral contraceptive prescription. She has no problems today and appears fit and well. On palpation of the thyroid, the right lobe is small, smooth, and free of nodules, but appears to be slightly larger than the left lobe. She has no other unexpected physical findings. The enlargement most likely represents which of the following? A inflammation of the thyroid B goiter C hypothyroidism D malignancy E normal finding

E normal finding Slight enlargement of one lobe of the thyroid gland in a healthy woman with no complaints is most likely a normal finding. In patients with an inflammation of the gland (A), i.e., thyroiditis, and hypothyroidism (C) the thyroid is usually diffusely enlarged and may be markedly asymmetric. Other findings depend upon the cause. Goiters (B) are typically nodular and may be quite enlarged. Thyroid malignancy (D) usually presents as a firm, non-tender nodule.

A 49-year-old man presents to the office complaining of general malaise with muscle aches, anorexia, fever, and severe pain over his anterior neck radiating to his ears. He states that he was ill about 2 weeks ago with a sore throat, but it resolved within a few days. On palpation, the thyroid gland is enlarged and tender. His laboratory workup shows a high T4 level and increased erythrocyte sedimentation rate (ESR). What is the most appropriate therapy for this patient's disease? A levothyroxine sodium B PTU therapy C radioiodine ablation D surgery E supportive therapy only

E supportive therapy only This is subacute, painful thyroiditis. This is a self-limiting disorder that at most requires symptomatic therapy. In mild cases, analgesics (ASA) are sufficient for pain relief and to decrease the inflammation. Prednisone may bring more relief if needed. Transient hypothyroidism should be treated as well.

A 65-year-old woman presents to the office with decreased hearing, and pain over her sternum, pelvis, and her right tibial tubercle. On x-ray, the involved bones are noted to be expanded and denser than normal. Her serum calcium and phosphorus levels are normal, but serum alkaline phosphatase level is markedly elevated. Which of the following would be the appropriate initial treatment for this patient? A ibuprofen 600 mg po every 6 hours B indomethacin 25 mg po tid C meclizine 25 mg po tid D methotrexate 7.5 mg po qd E tiludronate 400 mg po qd

E tiludronate 400 mg po qd This patient's signs and symptoms are consistent with Paget disease of bone. Biphosphates have become the treatment of choice for this disease. Tiludronate, taken orally for 3 months, is very effective in treatment of this disease.

A 42-year-old woman comes in with a "lump in her neck." When she was a teenager, she underwent radiation treatment for "some kind of tumor" in her neck. Ultrasound reveals a 1.5 cm lesion in the left lobe of the thyroid; biopsy confirms papillary carcinoma. Thorough evaluation reveals no evidence of metastasis. Which of the following is the initial treatment of choice for her? A chemotherapy B lobectomy with isthmectomy C radiation therapy D radioactive iodine therapy E total thyroidectomy

E total thyroidectomy Total or near-total thyroidectomy is indicated for this woman. No chemotherapy (A) is available for thyroid cancer. Lobectomy with isthmectomy is reserved for papillary carcinoma that is less than 1 cm in size in persons with no history of radiation exposure and no evidence of metastasis. Radiation therapy (C) is used to treat bone metastasis and anaplastic carcinoma. Radioactive iodine therapy (D) may be used following thyroidectomy to ablate any remnant of the gland and to treat cancer that has metastasized or is otherwise high risk.

A 42-year-old woman has a developed a solid and quite firm thyroid mass that is approximately 1 cm diameter by palpation. She does not have any hoarseness, difficulty breathing or swallowing, or symptoms of thyroid disease. She has never been exposed to radiation and has no history of cancer, nor does she have a family history of thyroid or other cancers. A thyroid panel is within normal limits. What is the next step in the care of this patient? A fine-needle aspiration biopsy of the nodule B observation to see if the nodule increases in size C radionuclide thyroid scan D resection of the nodule E ultrasonography of the neck

E ultrasonography of the neck The next step in the care of this woman is neck ultrasonography to determine its size and characteristics; this technique is preferred to CT or MRI scanning because it is accurate, noninvasive, far less costly. Should the ultrasound results be suspicious for malignancy, a fine-needle aspiration biopsy (A) should follow. A thyroid mass 1 cm in diameter or larger must be evaluated rather than observed (B). Radionuclide scanning (C) would be indicated if the patient's TSH were low but is otherwise limited in determining whether a thyroid mass is malignant. Resection (D) would be inappropriate until further testing is done.


संबंधित स्टडी सेट्स

Chapter 48: Nursing Care of the Child With an Alteration in Metabolism/Endocrine Disorder

View Set

PSYC330 - Abnormal Child Psyc - Exam 2

View Set

International Business Chapter 9

View Set